User Avatar
mlnewcomb936
Joined
Apr 2025
Subscription
Free
User Avatar
mlnewcomb936
Saturday, Oct 26 2019

Congrats!!! What helped you the most to improve this much?

User Avatar
mlnewcomb936
Sunday, Aug 19 2018

@, Awesome, I'm going to try to do that. I'm still in the window of the 14-day money back guarantee, so hopefully, they'll let me return the premiere package so I can go on and buy the Ultimate package and get it for longer/less money.

So, I bought an in-class LSAT prep package through Kaplan. However, after I paid/started the course, I started seeing all the negative reviews/couldn't get a refund for my Kaplan course and found 7sage, which has been really awesome. I'm going to finish out the Kaplan course (I'm halfway done), but once it ends, I want to hit the ground running fully switching over to 7sage.

I only have the premiere package for 7sage and it looks like most of the PrepTests provided, most of them will have been used in class materials from Kaplan. Is this an issue? I'm also planning on downloading all the PrepTests that come with my Kaplan course online so I can have access to them, but I won't have the 7sage explanations. Should I modify the preptests I do to be the ones that I won't have done with Kaplan, still do BR, but just independently look up answer explanations? Also, any advice from people who have made the switch from other LSAT prep courses and what to look out for?

User Avatar
mlnewcomb936
Friday, Jul 12 2019

Is there any chance the explanations for PT 87 will be available before July 15th? Thank you!!

Hey guys,

I've been having a lot of trouble understanding the explanation behind this game, which is also explained in the curriculum under sequencing games with a twist as the third example.

There can only be one repeating product on the board. For the rule "G is not advertised during a given week unless either J or else O is also advertised that week", JY says that we know that G cannot repeat b/c it'd carry another component with it. However, I don't understand why this is necessarily known, when G could repeat and bring J one time and bring O the second time. Am I missing something?

Thanks y'all!

https://classic.7sage.com/lsat_explanations/lsat-21-section-1-game-4/

https://classic.7sage.com/lesson/sequencing-game-wa-twist-3-game-board-setup/

User Avatar
mlnewcomb936
Tuesday, Dec 04 2018

#4) "Which one of the following principles must be assumed in order for the psychologist's conclusion to be properly drawn?"

It says that this is necessary assumption, but isn't this question sufficient assumption b/c it's asking for the assumption needed to ensure the argument is properly drawn/valid?

#help

Confirm action

Are you sure?